Discussion

Which one of the following, if true, most helps to support the claim made in the letter regarding the justification of the editorial’s conclusion?
(A)Departments other than the Planning Department have had much larger budget increases since 2001.
(B)...
(C)...
(D)...
(E)...
(F)...
*This question is included in Complete Section: LR-A, Sept. '09 LSAT (PT 58 Sec. 1), question #3

The solution is

Posted: 11/14/2012 14:12
Why is Answer B wrong?
Posted: 11/14/2012 18:06
Tyrone, answer B is not the most helpful in supporting the claim made in the letter. Even if there has been a dramatic reduction in overtime pay, there may be an equally or even more dramatic increase in other areas of the budget.

The claim is: it is not true that the department now spends 5x as much money as in 2001 to do the same amount of work. So the most helpful evidence would be anything that shows that the department does more work than in 2001, commensurate to the expansion in budget.

You need to be signed in to perform that action.

Sign In